0% found this document useful (0 votes)
43 views

Complex Analysis 1996

The document contains text extracted from a UPSC Civil Services Main 1996 paper on Complex Analysis. It provides solutions to 3 questions: 1) Sketching an ellipse described by a complex function and identifying positions of minimum velocity and maximum acceleration. 2) Proving every polynomial has exactly n roots using the fundamental theorem of algebra and Rouche's theorem. 3) Evaluating an integral using the residue theorem and finding the Laurent expansion of a function about a singularity.

Uploaded by

Suresh Kannan
Copyright
© Attribution Non-Commercial (BY-NC)
We take content rights seriously. If you suspect this is your content, claim it here.
Available Formats
Download as PDF, TXT or read online on Scribd
0% found this document useful (0 votes)
43 views

Complex Analysis 1996

The document contains text extracted from a UPSC Civil Services Main 1996 paper on Complex Analysis. It provides solutions to 3 questions: 1) Sketching an ellipse described by a complex function and identifying positions of minimum velocity and maximum acceleration. 2) Proving every polynomial has exactly n roots using the fundamental theorem of algebra and Rouche's theorem. 3) Evaluating an integral using the residue theorem and finding the Laurent expansion of a function about a singularity.

Uploaded by

Suresh Kannan
Copyright
© Attribution Non-Commercial (BY-NC)
We take content rights seriously. If you suspect this is your content, claim it here.
Available Formats
Download as PDF, TXT or read online on Scribd
You are on page 1/ 29

This document contains text automatically extracted from a PDF or image file.

Formatting may have been lost and not all text may have been recognized.
To remove this note, right-click and select "Delete table".

UPSC Civil Services Main 1996 - Mathematics

Complex Analysis
Sunder Lal
Retired Professor of Mathematics Panjab University Chandigarh

February 20, 2010


Question 1(a) Sketch the ellipse C described in the complex plane by z = Acost + iB sint,A > B where t is a real variable and A,B, are positive constants. If C is the trajectory of a particle with z(t) as the position vector of the particle at time t, identify with justication 1. the two positions where the velocity is minimum. 2. the two positions where the acceleration is maximum. Solution. We are given that x = Acost,y = B sint which implies that A2 x2 y2 Since A > B, it follows that it is the standard ellipse with 2A as the major axis + and B2 2B = as 1. the minor axis. B(0,B) C A (A,0) O A(A,0) B (0,B) 1

= A22 sin2 t + B22 cos2 t = (A2 B2)sin2 t + B2 Since A2B2 > 0, the speed is minimum when sin2 t = 0 i.e. when x(t) = A,y(t) = 0 i.e. when the particle is at the two ends of the major axis, the points A and A in the gure. 2. Acceleration = d2z dt2 = A2 Magnitude of acceleration = cost iB2 sint. 2 A2 cos2 t + B2 sin2 t = 2 (A2 B2)cos2 t + B2. Since A2 B2 > 0, acceleration is maximum when cos2 t = 1 cost = 1 i.e. the particle is at either end of the major axis, A or A . (Note that acceleration is minimum when cos2 t = 0 i.e. the particle is at either end of the minor axis). Question 1(b) Evaluate z0 lim 1 sin(z2) cosz . Solution.
z0

lim 1 cosz 2sin2 z


2 sin2 z (z 2 2

sin(z2) sin(z2) 2

4 1 2 Note that sinz has a simple zero at z = 0 and sinz = z(z) where (z) is analytic and (0) = 1, so z0 lim = z0 lim = z0 lim
)2 sin(z2) z2

= sinz z = 1. Question 1(c) Show that z = 0 is not a branch point for the function f(z) = sin z z . Is it a removable singularity? we Solution. x a branch We know of w = that z, w sin = z z is is a multiple valued function and has two branches. Once analytic, and sin z= z ( 3!

z)3 + ( 5! z)5 + ... or sin z z =1 3! z + z2 5! z3 7! + ... 2 1. The velocity v = dz dt = Asint + iBcost. Speed = magnitude of velocity = dz dt

Thus z0 lim sin z z = 1, so z = 0 is not a branch point of the function f(z) = sin z z . In fact z = 0 is a removable singularity of f(z). In fact F(z) = {
sin z z

,z=0 1, z = 0 is analytic everywhere once a branch of z is specied. Question 2(a) Prove that every polynomial equation a
0

z2 +...+a
n

zn = 0,a
n

= 0,n 1 has exactly n roots. Solution. Let P(z) = a


0

+a
1

z +a
2

for any z C. Let f(z) = P(z) +1

a
1

z,+a
2

z2 + ... + a
n

zn. Suppose, if possible, that P(z) = 0 then f(z) is an entire function i.e. f(z) is analytic in the whole complex plane. We shall now show that f(z) is bounded. P(z) = zn ) Since ( a
n

+ a
n1

z + a z2
n2

+ ... + zn a
0

a
j

znj 0 as z , for 0 j < n, is follows that given =


|a n | 2n

there exists R > 0 such that |z| > R a


j

< |a
n

| znj 2n for 0 j < n. Thus and therefore |f(z)| = a


n

+ a
n1

z + a
n2

z2 + ... + zn a
0

|a
n

|n

2n a
n

= a 2
n

1 P(z) = zn ( a
n

1
a n1 a

n2


z z2

+ + + ... + zn
a 0

) |a
n

2 |Rn for |z| > R Since |z| R is a compact set and f(z) is analytic on it, f(z) is bounded on |z| R. Consequently f(z) is bounded on the whole complex plane. Now we use Liouvilles theorem If an entire function is bounded on the whole complex plane, then it is a constant. Thus f(z) and therefore P(z) is a constant, which is not true, hence our assumption that P(z) = 0 for all z C is false. So there is at least one z
1

C where P(z
1

) = 0. (This result is called the fundamental theorem of algebra.) We now prove by induction one zero namely z = a
a 0 1

on n that P(z) has n zeros. If n = 1, P(z) = a


0

+a
1

z has . Assume as induction hypothesis that any polynomial of degree n1 has n1 zeros. By

Euclids algorithm, , we get P


1

(z) and R(z) such that P(z) = (z z


1

)P
1

(z) + R(z), where R(z) 0 or degR(z) < 1 i.e. R(z) is a constant. Putting z = z
1

we get R(z) 0, so P(z) = (z z


1

)P
1

(z). Since P
1

(z) is a polynomial of degree n 1, by induction hypothesis it has n 1 roots in C, and therefore P(z) has n roots in C. 3

,...,z

We now prove that P(z) has exactly n roots. Let z


1

,z
2 n

be the (not necessarily distinct) roots of P(z). Let g(z) = (z z


1

P(z) . Clearly g(z) is analytic in the whole complex plane. Since


z

lim )(z z
2

)...(z z
n

) g(z) = z lim P(z) (z z


1

)(z z
2

)...(z z
n

) = a
n

+
a n1 z

+
a n2 z2

+ ... + zn
a 0

(1 z
z

)(1 z
z 2

)...(1 z
z n

) =a
n

it follows that given > 0 there exists R such that |g(z) a


n

| < for |z| > R, so g(z) is bounded in the region |z| > R. The function g(z) being analytic is bounded in the compact region |z| R. Thus by Liouvilles theorem g(z) is a constant, in fact g(z) = a
n

, and therefore P(z) = a


n

(z z
1

)(z z
2

)...(z z
n

) Thus if is a zero of P(z), then = z


j

for some j, 1 j n. Thus P(z) has exactly n zeroes. Alternate Proof: We shall use Rouches theorem Let be a simple closed rectiable curve. Let f(z),g(z) be analytic on and within . Suppose |g(z)| < |f(z)| on , then f(z) and f(z) g(z) have the same number of zeroes inside . Let f(z) = a
n

zn and g(z) = a
n1

zn1 + ... + a
0

. Let R be so large that |g(z)| < |f(z)| on |z| = R. Then f(z) and f(z) + g(z) = P(z) have the same number of zeroes within |z| = R. But whatever R > 0 we take, f(z) has exactly n zeroes in |z| = R, therefore P(z) has exactly n zeroes in C.

Note: Rouches theorem follows from the Argument Principle Note that

(arg(f(z)+ g(z))) =change in argument of f(z)+g(z) as z moves along =

argf(z)+

arg(1+
f(z) g(z)

) as f(z) = 0 along . But

arg(1+
f(z) g(z)

) = 0 because |
f(z) g(z)

| < 1 and therefore


g(z) f(z)

continues to lie in the disc |w 1| < 1 as z moves on i.e. does not go around the origin. Question 2(b) By using the residue theorem, evaluate
0

log
e

(x2 x2 + + 1 1) dx Solution. Let f(z) = log(z log(z + i) in C + 1 {z + z2 | i) and z = we consider iy,y 1}, where it is single-valued. Let be the contour consisting of the line joining (R,0) and (R,0) and , which is the arc of the circle of radius R and center (0,0) lying in the upper half plane. is oriented counter-clockwise.

(R,0) (0,0) (R,0) 4

Clearly f(z) has a simple pole at z = i in the upper half plane. The residue at z = i is

lim
zi

(z + i)log(z 1 + z2 + i) = log2i 2i = 2i 1 log2e


i 2

= 2i 1 [ log2 + i 2 ] = 4 1 2 ilog 2 Thus by Cauchys residue theorem


R

lim

log(z 1 + + z2 i) = R lim

log(z 1 +

[ ] 4 as z = x on the real axis. We shall now show that R lim + z2 i) + log(x 1 + + x2 i) dx = 2i 1 2 ilog2

log(z

1 + + z2 i) = 0. On , z = Rei, so log(z + i) 1 + z2 =

log(Rei R2e2i + + i)Riei 1 d Now |R2e2i + 1| |logR + i| logR

R2 1,log(Rei + i) = logRei + log(1 + i


Rei

). Clearly |logRei| = + log(z and therefore 1 + + z2 i)


0

( + logR)R R2 1

d +
0

R|log(1 + i
Rei

R2 1 )| d Since

( + R2 log R)R 1

log(z 0. 0 and
R|log(1+ R21 Re i )|

0 as R , it follows that R lim 1 + + z2 i) = Thus


i

2 2 Equating

real and imaginary parts, we get


0

log(x 1 + + x2 i) dx = log2 + i log(1 1 + + x2 x2) dx = 1 2


log(1 1 + + x2 x2) dx = 1 2

log(x + i) 1 + + x2 log(x i) dx = 1 2 [2log 2] = log2 Question 2(c) Find the Laurent expansion of f(z) = (z 3)sin ( z+1 2 ) about the singularity z = 2. Specify the region of convergence and the nature of the singularity at z = 2. Solution. It is well known that sin ( z+1 2 ) = ( 1 z+2 )
2k1

(z 3)sin
k=1

(1)k1 (2k 1)! ( 1 z+2 ) = (z + 2)sin ( z+1 2 ) 5sin

( z+1 2 ) = (z + 2) ( 1 z+2 )
2k1

5 ( 1 z+2 )
2k1

=
k=1

(1)k1 (2k 1)!


k=1

(1)k1 (2k 1)!


k=0

(z + a
k

2)k ,a
2k2

= (2k (1)k1 1)! ,a


2k1

= 5(1)k1 (2k 1)! 5

The region of convergence of the series is 0 < |z + 2| < . The Laurent expansion shows

that the function has an essential singularity at z = 2 this also follows from the fact that lim
z0

sin 1
z

does not exist. 6

You might also like